You are on page 1of 46

University

of London EC2066 (Andy Lee) 1


Production, costs and profit maximization


Essential reading
Subject guide chapter 5
N&S Chapters 6 and 7, Sections 8.1 and 8.2, and Sections 13.1 and 13.2.

Introduction
Over the next few chapters we develop models to analyse firms operating in different
market structures. In particular, we analyse two extremes: a competitive market and a
monopoly.

Later on we also analyse oligopoly. To develop these models, we need to make use of
a variety of concepts on the nature of technology, costs and optimisation by firms.

Production and factor demand


The production function of a firm records the maximum output from a given set of
inputs (capital and labour). It is a mathematical relationship between inputs and
outputs.

Capital (such as plants and machinery) is assumed to be fixed in the short run and can
only be varied in the long run.

Labour on the other hand can be varied in both the short run and long run.

The production function with two inputs can be written as a function:

Q = f(K, L).

Marginal product
Marginal product of labour (𝑀𝑃! ) can be defined as the additional output that can be
produced by adding one more unit of labour while holding capital constant.

In mathematical terms, it is the partial derivative1 of the production function with


respect to labour.

𝜕𝑓 𝐾, 𝐿
𝑀𝑃! =
𝜕𝐿


1In mathematical terms, the 𝑀𝑃! is the derivative of the production function with respect to L. Because
K is held constant in defining the 𝑀𝑃! , this derivative is a ‘‘partial’’ derivative.


University of London EC2066 (Andy Lee) 2

As an illustration, consider the case of a farmer hiring one more person to harvest a crop while holding
all other inputs constant. The extra output produced when this person is added to the production team is
the marginal product of labor.

For example, suppose we observe that 25 workers in an orange grove are able to produce 10,000 crates
of oranges per week, whereas 26 workers (with the same trees and equipment) can produce 10,200
crates. The marginal product of the 26th worker is 200 crates per week.

Similarly, the marginal product of capital (𝑀𝑃! ) can be defined as the additional
output that can be produced by adding one more unit of capital while holding labour
constant.

In mathematical terms, it is the partial derivative2 of the production function with


respect to capital.

𝜕𝑓 𝐾, 𝐿
𝑀𝑃! =
𝜕𝐾


2 In mathematical terms, the 𝑀𝑃 is the derivative of the production function with respect to K.
!
Because L is held constant in defining the 𝑀𝑃! , this derivative is a ‘‘partial’’ derivative.


University of London EC2066 (Andy Lee) 3

Diminishing marginal product of labour (in the short run)


Recall that K is fixed in the short run.

Suppose now, we increase our workers in the orange grove while holding the capital (machines,
equipment, fertilizer and so forth) constant. What do you think will happen to the 𝑀𝑃! ?

Ans: As the fixed amount of capital becomes overutilized, the 𝑀𝑃! will diminish.

This possibility is illustrated in Figure below.

Geometrically, the slope of the total output curve is the marginal product.

The decreasing slope of the total output curve shows diminishing marginal product of labor. (as shown
in panel b).

Therefore, the concave shape of the total output curve in panel (a) reflects the law of diminishing
marginal product.

Note also that at 𝐿∗ additional labor input does not raise total output at all.

This may happen because he or she has nothing useful to do in an already crowded orange
grove. The marginal product of this new worker is therefore zero.


University of London EC2066 (Andy Lee) 4

Mathematically, in the region with diminishing marginal product of labor, we have:

𝜕 ! 𝑓 𝐾, 𝐿
<0
𝜕𝐿!

We can apply the similar concepts to 𝑀𝑃! . Therefore, in the region with diminishing marginal
product of capital, we have :

𝜕 ! 𝑓 𝐾, 𝐿
<0
𝜕𝐾 !

Average product
The average product of labour (output per worker) is given by:

𝑄
𝐴𝑃! =
𝐿

In our orange grove example, 25 workers can produce 10,000 crates of oranges per week. So 𝐴𝑃! is
400 crates of oranges per week

Relationship between average product and marginal product


What is the relationship between average and marginal product? You should
understand the general relationship between average and marginal. Suppose you play
a computer game and your average score is 60. The next time you play, you score 100.
What will happen to your average score? It will go up. Here the marginal score
exceeded the average, so the average goes up.

Similarly, if the marginal were below the average, the average would fall.

The same principle applies to average and marginal product (and to other
comparisons such as between average and marginal cost that we would see later).

The average product rises if the marginal is above the average.

The average product falls if the marginal is below the average.

It follows that the average product curve crosses the marginal product curve at the
highest point on the average product curve. In other words, average product equals
marginal product when average product is at the maximum.


University of London EC2066 (Andy Lee) 5

The numerical example below demonstrates this (assuming that in the short run, K is
fixed at some positive level, say maybe 4)

L Q 𝐴𝑃! 𝑀𝑃!
1 10 10 10
2 25 12.5 15
3 45 15 20
4 68 17 23
5 90 18 22
6 108 18 18
7 121 17.29 13
8 126 15.75 5
9 126 14 0
Note that in this example, 𝑀𝑃! is increasing at first due to specialization (by
increasing the number of workers, they can specialize more in what they do and the efficiency
of the workers increases).

However, 𝑓𝑢𝑟𝑡ℎ𝑒𝑟 𝑖𝑛𝑐𝑟𝑒𝑎𝑠𝑒𝑠 𝑖𝑛 𝑤𝑜𝑟𝑘𝑒𝑟𝑠 𝑟𝑒𝑠𝑢𝑙𝑡𝑠 𝑖𝑛 𝑀𝑃! falling. This is because


there is not enough K for the workers to work on. (resulting in diminishing marginal
product of labour).


University of London EC2066 (Andy Lee) 6

!"! ! !"!! !!!


𝐴𝑃! = Q/L = !
= 10 + 30𝐿 − 𝐿!

!"
𝑀𝑃! = !"
= 10 + 60𝐿 − 3𝐿!

𝑀𝑃! is maximized when the derivative of 𝑀𝑃! with respect to L equals zero

!"!! !!!
!"
= !!!
=0

𝑑𝑀𝑃! 𝑑 ! 𝑄
= ! = 60 − 6𝐿 = 0
𝑑𝐿 𝑑𝐿

L = 10.

So 𝑀𝑃! is maximized at L = 10.

𝐴𝑃! is maximized when the derivative of 𝐴𝑃! with respect to L equals zero.

𝑑𝐴𝑃!
= 30 − 2𝐿 = 0
𝑑𝐿
L = 15

So 𝐴𝑃! is maximized when L=15. Therefore, 𝐴𝑃! curve crosses 𝑀𝑃! curve at L=15.

This is shown in the figure below.


University of London EC2066 (Andy Lee) 7

L
Figure 5.1


University of London EC2066 (Andy Lee) 8

The short run demand for Labour


In the short run, capital is fixed, only labour is variable. How should a firm choose
how much labour to hire, and how does the level of hiring vary when the wage rate
varies?

Below, we construct a numerical example to examine the hiring decision of the firm
for labour in the short run.

K L Q 𝑀𝑃! 𝑀𝑅𝑃! W
1 1 20 20 30 7
1 2 30 10 15 7
1 3 37 7 10.5 7
1 4 42 5 7.5 7
1 5 44 2 3 7

The table above shows a firm’s variable labour input and corresponding output, holding capital
input fixed (say at 1). Column (4) shows the marginal product of labour which is diminishing due
to the fixed K.

Suppose this is a competitive firm hiring workers at a wage of £7 (W = £7) and selling
output at a price of £1.50. Recall from EC1002 that for a competitive firm, price = marginal
revenue, P=MR. So in this example, P=MR=£1.50

Recall from EC1002 that MR is the extra revenue the firm gets for selling one more unit of
output. Since in this example, K is fixed, so output and hence revenue can only increase by
increasing labour. So we can define 𝑀𝑅! as the extra revenue the firm gets for selling one
more unit of output when it increases labour. In this example, 𝑀𝑅! = 𝑃 =£1.50

Also , note that hiring one more extra labour raises output by 𝑀𝑃!

Thus. the extra revenue the firm gets for hiring one more extra labour can be found by
multiplying 𝑀𝑃! with 𝑀𝑅! .

This extra revenue3 that the firm gets for hiring one more extra labour is known
as the 𝑴𝑹𝑷𝑳

Therefore, 𝑀𝑅𝑃! = 𝑀𝑃! × 𝑀𝑅!


3 Note that in the recommended textbook, N&S, 𝑀𝑅𝑃
! is referred to as 𝑀𝑉𝑃! to
highlight the special case of a competitive firm where MR=P


University of London EC2066 (Andy Lee) 9

Column (5) shows the 𝑀𝑅𝑃!

How many workers should the firm employ?

The firm will employ more labour if the extra revenue it gets from hiring one more extra
labour exceeds the wage cost of hiring that one more extra labour, that is 𝑀𝑅𝑃! exceeds the
wage cost.

In the example above, the firm will hire only up to 4 Labour.

The fifth labour’s 𝑀𝑅𝑃! is only £3; that is below the £7 another extra labour costs.
Therefore, if the firm hires the fifth worker, profits will be reduced.

So the firm should not hire him. It is optimal for the firm to hire only up to 4 labour.

We can therefore write down the firm’s employment rule.





1.The firm should expand employment if the 𝑀𝑅𝑃! is greater than the wage of an extra
labour.

2.The firm should contract employment if the 𝑀𝑅𝑃! is less than the wage of an extra labour.

3.It follows that the profit maximizing level of employment is found where the 𝑴𝑹𝑷𝑳 = W


University of London EC2066 (Andy Lee) 10

Figure 5.2 shows the 𝑀𝑅𝑃! curve. Note that it is downward sloping due to
diminishing marginal product of labour (since for the case of a competitive firm, MR
is constant at P, but 𝑀𝑃! falls as labour rises).

When the wage is 𝑤! , the firm optimally employs 𝐿! workers. When the wage falls to
𝑤! , the firm optimally employs 𝐿! workers. Therefore, for a competitive firm with a
downward sloping 𝑀𝑅𝑃! , more labour will be hired when wage rate falls.

Furthermore, we can conclude that the 𝑀𝑅𝑃! curve is the competitive firm’s short run
labour demand curve when K is fixed.


University of London EC2066 (Andy Lee) 11

The long run demand for inputs (K and L)


In the long run, K and L are variable.

With two variable factors (in the long run), we can represent the production function
by its level curves.

To show the various combinations of capital and labor that can be employed to produce a particular
output level, we use an isoquant (from the Greek iso , meaning ‘‘equal’’).

Figure 6.2

For example, all the combinations of K and L that fall on the curve labeled q = 6 in Figure 6.2 are
capable of producing 6 units of output.

This single isoquant records the many alternative ways of producing 6 units of output. One
combination is represented by point b. A firm could use 12 capital and 3 labor to produce 6 units of
output.

Alternatively, the firm might prefer to use relatively less capital and more labor and would therefore
choose a point such as e (with 3 capital and 12 labor).

Note that isoquants further to the right represents higher output since there are more inputs. To see why,
consider point g and point b. Point b has 12 capital and 3 labor that is capable of producing 6 output.
Point g also has 12 capital but more labor (12 labor) Therefore, point g must represent an input
combination associated with a higher level of output than 6 (in this case 12)


University of London EC2066 (Andy Lee) 12

Marginal rate of technical substitution (MRTS)


The marginal rate of technical substitution of labour for capital ( 𝑀𝑅𝑇𝑆!" ) is defined as
the amount of capital input that can be reduced when one more unit of labor input is increased while
holding output constant.

Δ𝐾
Therefore, the 𝑀𝑅𝑇𝑆!" can be defined as −
Δ𝐿

For example, in figure 6.2, as seen from point a to point b, the firm can give up 6 capital (Δ𝐾 = −6) in
exchange for 1 labor (Δ𝐿 = 1) and still produce the same output. Therefore, 𝑀𝑅𝑇𝑆!" = - (-6 / 1) = 6

Δ𝐾
Notice that if we drew a straight line from a to b , its slope would be
Δ𝐿
= −6
Thus, this slope tells us how many fewer units of capital (6) the firm can use if it hires 1 more labor. 4

We conclude that the negative of the slope of isoquant is the 𝑀𝑅𝑇𝑆!"

𝑴𝑹𝑻𝑺𝑳𝑲 is the ratio of marginal products


As we move along to the right of the isoquant, the firm hires more labor and less capital.

Now, recall that the marginal product of labor, 𝑀𝑃! = ΔQ/ΔL, is the increase in output
per extra unit of labor.

So if the firm hires more workers, its output increases by 𝑀𝑃! x (Δ L). For example, if the 𝑀𝑃! is 2
and the firm hires one extra worker, its output rises by 2 units.


4 The slope of the isoquant at a point equals the slope of a straight line that is tangent to the isoquant at that point. Thus,
the straight line between two nearby points on an isoquant has nearly the same slope as that of the isoquant


University of London EC2066 (Andy Lee) 13

Thus, as we move to the right along the isoquant, output increases by 𝑀𝑃! x (Δ L)

However, as we move to the right along the isoquant, the firm also hires less capital. Recall that the
marginal product of capital, 𝑀𝑃! = Δ Q/Δ K. Therefore, output falls by 𝑀𝑃! x (Δ K) as we move to
the right along the isoquant.

Since output is constant along an isoquant, this fall in output from reducing capital must exactly equal
the increase in output from increasing labor:

(𝑀𝑃! x ΔL ) + (𝑀𝑃! x ΔK ) = 0.

Rearranging these terms, we find that

Δ𝐾

Δ𝐿
= 𝑀𝑃! / 𝑀𝑃!
𝑀𝑅𝑇𝑆!" = 𝑀𝑃! / 𝑀𝑃!

That is, the marginal rate of technical substitution, which is the change in capital relative to the change
in labor, equals the ratio of the marginal products.


University of London EC2066 (Andy Lee) 14

Diminishing 𝑴𝑹𝑻𝑺𝑳𝑲
In the figure below, from point a to b, 𝑀𝑅𝑇𝑆!" = 6. Moving from point b to c, its 𝑀𝑅𝑇𝑆!" is 3

If it adds yet another worker, moving from c to d, its 𝑀𝑅𝑇𝑆!" is 2

Finally, if it moves from d to e, its MRTS is 1

Thus, the isoquant exhibits a diminishing marginal rate of technical substitution ( giving the isoquant a
convex shape)

That is, each extra worker allows the firm to reduce capital by a smaller amount as the ratio of capital
to labor falls.

Why does the isoquant exhibits diminishing 𝑴𝑹𝑻𝑺𝑳𝑲 ?


When capital is plentiful and labour is scarce (point a), the marginal product of capital relative to
labour is low - when there are many pieces of equipment per worker, each remaining piece is less
useful.

Thus, when capital is plentiful and labour is scarce (point a) the firm can afford to give up a lot of
capital in return for a little more of the scarcer input. So 𝑀𝑅𝑇𝑆!" is higher (isoquant is steeper)

However, when labour is plentiful and capital is scarce (point e), the reverse is true. The marginal
product of capital relative to labour is high - when there are few pieces of equipment per worker, each
remaining piece is more useful.


University of London EC2066 (Andy Lee) 15

Thus, when labour is plentiful and capital is scarce (point e) the firm can only afford to reduce capital
by a small amount in return for more labour. So 𝑀𝑅𝑇𝑆!" is smaller (isoquant is flatter)

So 𝑀𝑅𝑇𝑆!" falls as we move to the right along the isoquant.

No input substitution
In the discussion above, we have assumed a production function where the firm can substitute one
input for another.

If inputs cannot be substituted at all, how will the isoquant look like?
For example, the inputs needed to produce a box of cereal (q), are cereals (g), and cardboard boxes (b).

This fixed-proportions production function is

q = min(g , b)

where the min function means “the minimum number of g or b .”

For example, if the firm has g = 2 units of cereal and b = 1 box, it can produce only q = 1
boxes of cereal. This implies that isoquants will be L-shaped.

Thus, the only efficient points of production are at the kink, where g=b, so that the firm uses equal
quantities of both inputs.


University of London EC2066 (Andy Lee) 16

Returns to scale
So far, we have examined the effects of increasing one input while decreasing the other input
by an offsetting amount (the movement along an isoquant).

We now turn to the question of how much output changes if a firm increases all its inputs
proportionately.

The answer helps a firm determine its scale or size in the long run.

In the long run, a firm can increase its output by building a second plant and staffing it with the same
number of workers as in the first plant.

Whether the firm chooses to do so depends in part on whether its output increases less than in
proportion, in proportion, or more than in proportion to its inputs.

For example, suppose all inputs were doubled. Would output also double, more than double, or less
than double?

A production function is said to exhibit constant returns to scale if a doubling of all inputs results in a
precise doubling of output.

If a doubling of all inputs yields less than a doubling of output, the production function is said to
exhibit decreasing returns to scale .

If a doubling of all inputs results in more than a doubling of output, the production function exhibits
increasing returns to scale

Many production functions have increasing returns to scale for small amounts of output, constant
returns for moderate amounts of output, and decreasing returns for large amounts of output.


University of London EC2066 (Andy Lee) 17

With a small firm (and hence a small amount of outputs), increasing labor and capital may produce
gains from cooperation between workers and greater specialization of workers and equipment—returns
to specialization —so there are increasing returns to scale.

As the firm grows (produce a moderate amount of output), returns to scale are eventually exhausted.
With no more returns to specialization, the production process exhibits constant returns to scale.

If the firm continues to grow (produce a large amount of output), managing the staff becomes more
difficult, so the firm suffers from decreasing returns to scale.

The figure below shows such a pattern. The spacing of the isoquants reflects the
returns to scale.

Initially, the firm has one worker and one piece of equipment, point a , and produces one unit of output
on the q = 1 isoquant.

If the firm doubles its inputs, it produces at b , where L = 2 and K = 2, which lies on the dashed line
through the origin and point a . Output more than doubles to q = 3, so the production function
exhibits increasing returns to scale in this range.

Another doubling of inputs to c causes output to double to q = 6, so the production function has
constant returns to scale in this range.

Another doubling of inputs to d causes output to increase by only one-third, to q = 8, so the production
function has decreasing returns to scale in this range.


University of London EC2066 (Andy Lee) 18


University of London EC2066 (Andy Lee) 19

Isocost
Isocost means equal cost. It is very similar to the budget line that we learnt in consumer choice.

The firm hires L hours of labor services at a wage of w per hour, so its labor cost is wL.

The firm rents K hours of machine services at a rental rate of r per hour, so its capital cost is rK.

The firm’s total cost is the sum of its labor and capital costs:

C = wL + rK

The isocost is drawn below. If the firm employs only capital, it can employ C/r units of capital.

If the firm employs only labour, it can employ C/w units of labour. The firm can choose to employ any
combinations of K and L that lies on the isocost.

Note that the slope of the isocost (in absolute terms) is w/r . This is known as the factor price ratio.

C/r

C/w L


University of London EC2066 (Andy Lee) 20

Numerical example
Exercise: Suppose r =2, w=4. Draw two different isocost for when cost is £40 and when cost is £100.
What can we say about an isocost that lies further from the origin?

Ans : Isocost that lies further from the origin implies higher cost (see diagram below).

Change in wage
Suppose w rises, what happens to the isocost?

Ans: The isocost pivots to the left and becomes steeper (see diagram below).


University of London EC2066 (Andy Lee) 21

Cost minimization
The firm will choose an input combination such that the cost is minimized. In other words, the firm
minimizes its cost by using the combination of inputs on the isoquant that lies on the lowest isocost line
to touch the isoquant.

This is shown below. The firm chooses the input bundle (K* and L*) where the isoquant is tangent to
the isocost line (𝐶! ) to produce the output Q at the lowest cost.

At the point of tangency, the slope of the isoquant equals the slope of the isocost.

Therefore, when cost is minimized,

𝑤
−𝑀𝑅𝑇𝑆𝐿𝐾 =−
𝑟
𝑤
𝑀𝑅𝑇𝑆!" =
𝑟
𝑀𝑃! 𝑤
=
𝑀𝑃! 𝑟


University of London EC2066 (Andy Lee) 22

Let us show this formally using a lagrange multiplier approach below.

Note that the cost minimizing condition can also be written as

𝑀𝑃! 𝑀𝑃!
=
𝑤 𝑟

The equation above is known as the last-dollar rule : Cost is minimized if inputs are chosen so
that the last dollar spent on labor adds as much extra output as the last dollar spent on capital.

Average cost and marginal cost (long run)


Suppose TC(Q) is the long run total cost function for a firm.

Average cost (AC) measures total costs per unit. Mathematically,the long run average cost is:

𝑇𝐶(𝑄)
𝐴𝐶 =
𝑄

For example, if a firm has total costs of $100 in producing 25 units of output, then average cost is $4.

Note that, geometrically, AC is the slope of a chord from the origin to the relevant point in
the TC curve


University of London EC2066 (Andy Lee) 23

For economists, however, average cost is not the most meaningful cost-per-unit figure. Economists also
use the concept of marginal cost (MC). By definition, the long run marginal cost is:

∆!"(!) 𝑑𝑇𝐶(𝑄)
MC = ∆!
= 𝑑𝑄

For example, if producing 24 units costs the firm $98 but producing 25 units costs it $100, the marginal
cost of the 25th unit is $2: To produce that unit, the firm incurs an increase in cost of only $2.
This example shows that the average cost of a good ($4) and its marginal cost ($2) may
be quite different.

Note that geometrically, the MC is the slope of TC curve.


University of London EC2066 (Andy Lee) 24

Exercise: Explain why MC is the slope of TC curve and why AC is the slope of the chord
from the origin to the TC curve


University of London EC2066 (Andy Lee) 25

Long run cost curves


What is the relationship between output, long run total costs, average costs and
marginal costs?

That depends on the nature of production - whether the firm is experiencing constant,
decreasing or increasing returns to scale

Below, we examine the 3 different cases.

With constant returns to scale, output and required input use are proportional to one
another. A doubling of output requires a doubling of inputs. If input prices
do not change, the relationship between output and total input costs is also directly
proportional—the total cost curve is simply a straight line that passes through the
origin (since no inputs are required if q = 0).

As noted above, marginal costs are reflected by the slope of the total cost curve. Since the total cost
curve is a straight line—it has the same slope throughout. In this case, the long run marginal cost (MC)
is constant. No matter how much is produced, it will always cost the same to produce one more unit .
The horizontal MC curve below reflects this fact.

Since marginal cost is constant, it always costs the same amount to produce one more
unit, and hence AC must also reflect this amount5.

This implies that both the AC and the MC curves are the same horizontal line as shown below.


5 For example, ifit always costs a firm the same amount of $4 to produce one more unit of the good,
(MC = 4), then average cost is also constant at AC = 4


University of London EC2066 (Andy Lee) 26

With decreasing returns to scale, successively larger quantities of inputs are required to increase output
and input costs rise rapidly as output expands.
This possibility is shown by the convex total cost curve below. In this case, costs expand more rapidly
than output.


University of London EC2066 (Andy Lee) 27

With increasing returns to scale, on the other hand, successive input requirements decline
as output expands. In that case, the total cost curve is concave, as shown below. Costs expand less
rapidly than output.

U- shaped AC and MC
How will AC and MC look like for a typical firm that exhibits increasing returns to scale followed by
decreasing returns to scale?

As we have discussed above, most production functions have increasing returns to scale for small
amounts of output and decreasing returns for large amounts of output. Therefore, the typical long
run total cost curve will be concave in its initial section and convex in the later section. This is shown
below.

Furthermore, since K and L are variable in the long run (in other words, in the long run, the firm, the
firm can vary its plant size (K) , and vary its labor), there is no fixed cost in the long run. Thus,
the long run total cost of producing zero output is zero. This is also shown in the diagram
below.


University of London EC2066 (Andy Lee) 28

Since AC = 𝑇𝐶(𝑄)/𝑄, it follows that AC falls at first (for low levels of output/quantity) when the firm
is experiencing increasing returns to scale Doubling inputs more than doubles output, so average cost
falls.

However, AC starts to rise later (for high levels of output/quantity) when the firm experiences
decreasing returns to scale. Doubling inputs less than doubles output, so average cost rises.

This implies that the long run AC will be U-shaped as shown in the diagram below.


University of London EC2066 (Andy Lee) 29

Let us now draw the AC and MC together. Do you still recall the relationship between average and
marginal?

Ans: Whenever marginal is below average, average must fall. Whenever marginal is above average,
average must rise.

Therefore, when MC is below AC, AC decreases.

When MC is above AC,AC increases.

It follows that MC and AC cross at the point where AC reaches a minimum. This is shown in
the diagram below


University of London EC2066 (Andy Lee) 30


University of London EC2066 (Andy Lee) 31

Short run total cost (SRTC)


How about total cost in the short run? In the short run, capital is fixed, but labour is variable.

Because capital input is held fixed in the short run, the costs associated with that input are also fixed.
This is the fixed cost. Note that fixed cost cannot be avoided by reducing output and must be incurred
as long as the firm stays in business. So fixed cost does not vary with the level of output.

On the other hand, costs associated with inputs that can be varied (labor in this case) are called variable
cost . The amount of these cost obviously will change as the firm changes its labor input so as to bring
about changes in output. In other words, variable cost depends on output. Higher output produced
implies higher variable cost.

Thus, we have:

SRTC(Q) = F + VC(Q)

where F is fixed cost, VC(Q) is variable cost.

It follows that:
! !"(!)
SRTC(Q) / Q = ! + !

! !"(!)
SRAC(Q) = ! + !

The first component is the average fixed cost (AFC), and the second is average
variable cost (AVC).

Thus, SRAC(Q) = AFC + AVC

The SRTC is drawn below. It is concave in its initial section (due to increasing 𝑀𝑃! ) and convex in the
later section (due to diminishing 𝑀𝑃𝐿 ). Note that the SRTC is positive even when output is zero due
to the fixed cost.

The fixed costs F is horizontal as it does not vary with output. In other words, the amount of capital
cost that the firm incurs is the same no matter how much output the firm produces - it must pay the
same rent on its fixed number of machines regardless of how much output it produces.

The vertical distance between the SRTC and VC is the fixed costs F, so the SRTC and VC are parallel.
Note that the variable cost curve is zero when output is zero since the firm does not need to hire labor if
it produces nothing.


University of London EC2066 (Andy Lee) 32

Let us now derive the SRAC, SRMC and AVC curves using the SRTC and VC curves.

As before, geometrically,
the SRAC is the slope of a chord from the origin to the SRTC
𝑑𝑆𝑅𝑇𝐶
curve and the SRMC = 𝑑𝑄 is the slope of SRTC curve.

Note also that the AVC is the slope of a chord from the origin to the VC curve.

Therefore, given that SRTC is concave in its initial section and convex in the later section, this
implies that the SRAC, AVC and SRMC will be U-shaped (see diagram below).

Also, SRMC curve will intersect SRAC curve when SRAC is at its minimum point. SRMC will also
intersect AVC curve when AVC curve is at its minimum.


University of London EC2066 (Andy Lee) 33

Furthermore, since SRAC – AVC = AFC, therefore, the gap between SRAC and AVC is the
AFC.
!
Since AFC = , so if Q rises, AFC falls. Thus, the gap between SRAC and AVC gets smaller
!
as Q rises.


University of London EC2066 (Andy Lee) 34

Exercise: Using the information above, derive the SRAC, SRMC and AVC curves graphically
from the SRTC and VC curves.


University of London EC2066 (Andy Lee) 35

Example 5.2 Plot the SRAC, AVC and SRMC for the short run cost curve:

SRTC(Q) = 150Q - 20𝑸𝟐 + 𝑸𝟑 + 500


University of London EC2066 (Andy Lee) 36


University of London EC2066 (Andy Lee) 37

The relationship between short-run and long-run


average costs
We have already learnt the short run and long run average costs. Let us bring them together now.

Recall that in the short run, K is fixed, therefore, the short run production is inferior compared
to the long run production due to the inflexibility in adjusting K.

Hence, the firm’s short-run total costs can never be lesser than its long-run total costs.

It follows that the the SRAC can never be lesser than the LRAC. This is shown in the diagram
below.

Suppose, initially, that the firm in Figure above has only three possible plant sizes.

The firm’s short-run average cost curve is 𝑆𝑅𝐴𝐶! for the smallest possible plant, 𝑆𝑅𝐴𝐶! for medium
plant size and 𝑆𝑅𝐴𝐶! for biggest plant size.

From the figure, we see that the average cost of producing 𝑞! units of output using the smallest plant
shown on point a on 𝑆𝑅𝐴𝐶! is $10.


University of London EC2066 (Andy Lee) 38

If instead the firm used the medium plant size, its cost of producing 𝑞! units of output, point b on
𝑆𝑅𝐴𝐶! would be $12.

Thus, if the firm knows that it will produce only 𝑞! units of output, it minimizes its average cost by
using the smaller plant size.

Exercise: If the firm wishes to produce 𝑞! , then which plant size should the firm use to minimize
its cost?

Answer: The firm should use the medium plant size and produce on 𝑺𝑹𝑨𝑪𝟐 . Point e is below
point d. So average cost is lower if the firm used the medium plant size rather than the small
plant size.

In the long run, the firm chooses the plant size that minimizes its cost of production, so it picks the
plant size that has the lowest average cost for each possible output level.

At 𝑞! , it opts for the small plant size, whereas at 𝑞! , it uses the medium plant size.

This argument can be repeated for other plant sizes.


.

If there are many possible plant sizes, the long-run average cost curve, LRAC, is smooth and U-shaped.
The LRAC includes one point from each possible short-run average cost curve


University of London EC2066 (Andy Lee) 39


University of London EC2066 (Andy Lee) 40


University of London EC2066 (Andy Lee) 41

Profit maximization
The profit of a firm is given by revenue minus cost

𝜋 = 𝑇𝑅 − 𝐶(𝑄) = 𝑃(𝑄) 𝑄 − 𝐶(𝑄).

This is maximised at the quantity where marginal revenue equals marginal cost:

Proof
The first order condition for profit maximization is:
!!
=0
!"

𝑑 𝑑𝐶 𝑄
𝑃 𝑄 𝑄 − =0
𝑑𝑄 𝑑𝑄

MR – MC = 0

MR=MC


University of London EC2066 (Andy Lee) 42

Note however, that for a price-taking firm, the price P does not depend on output since it is a
!(! !)
price taker, so = 𝑃 and profit is maximised at the output where
!"

P – MC = 0
P = MC

Intuition behind the MR=MC rule for profit maximization


Recall that Marginal cost (MC) is the change in total economic cost due to the production of one more
unit of output.

‘Marginal revenue’ (MR) is the change in total revenue due to the sale of one more unit of output.

Suppose MR>MC, then by increasing output, the extra revenue that the firm gets for selling the one
more unit of output exceeds the extra cost it incurs to produce that one more unit of output.

Therefore, by increasing output, the firm can increase its profits. We therefore conclude that if
MR>MC, profits are not yet maximized.

Suppose MR<MC, if the firm increases its output, the extra revenue that the firm gets for selling the
one more unit of output cannot cover the extra cost it incurs to produce that one more unit of output.

So by reducing output, the firm can increase its profits We therefore conclude that if MR<MC, profits
are not yet maximized.

So it must be the case that only when MR=MC, profits are maximized. We conclude that MR=MC is
the profit maximizing condition.


University of London EC2066 (Andy Lee) 43

To understand why a firm will shut down in the short run when P<AVC, let us
consider the example below.

Recall that SRTC = F +VC .

Profit is 𝜋 = TR – SRTC

Now, suppose TR >SRTC, thus profit >0.

Will the firm produce in the short run? Yes!

Now suppose TR<SRTC, thus profit <0.


Can we say that the firm will definitely not produce in the short run? No!

Why? Below, we use a numerical example to illustrate.


University of London EC2066 (Andy Lee) 44

Suppose the firm produces in the short run and receive TR = 10. The firm also incurs the following
costs. F = 5, VC = 7, so SRTC = 5 + 7 = 12

Therefore, profit = 10 - 12 = -2
So the firm incurs a loss of 2 if it produces.

If the firm shuts down, it will incur no variable cost and receive no revenue. In the short run, however,
the firm must still pay its fixed costs6 (for example, rent on its factory) even if it produces no output.

So, TR = 0, F= 5, VC = 0, so SRTC = 5
Profit = 0-5 = -5

So the firm incurs a loss of 5 if it does not produce. In other words, the firm loses the fixed cost if it
decides not to produce in the short run.

Therefore, the firm will still produce in the short run. Having a loss of 2 is better than having a loss of 5.

We can therefore, state down the decision rule for the firm.

A firm will produce in the short run if TR>VC

A firm will shut down in the short run if TR<VC

So we now know that a firm will shut down in the short run only if TR<VC

Dividing both sides by Q, we can also write down the shut down rule as

!" !"
<
! !

!!! !"
<
! !

P < AVC

In other words, a firm will shut down in the short run if P < AVC.

A firm will produce in the short run if P >AVC



6 We usually assume that fixed cost is a sunk cost. A sunk cost is a past expenditure

that cannot be avoided even if the firm stops operation. However, if a firm can sell
its capital for some money, part of its fixed cost is avoidable. For example,if a firm buys a specialized
piece of machinery for $1,000 that can be used only for its business but can be sold for scrap metal for
$100, then $100 of the fixed cost is avoidable and $900 is sunk. In this case, the firm will only lose
$900 (and not the entire fixed cost of $1000) if it shuts down in the short run.


University of London EC2066 (Andy Lee) 45

We can use some algebra to summarize our discussion above. We know short run total costs equal the
sum of fixed and variable costs:
SRTC = F +VC

and profits are given by


𝜋 = TR – SRTC = (P x Q) – (F +VC)

If Q = 0, variable costs and revenues are 0, so


𝜋=-F

The firm will produce something only if


𝜋>-F
(P x Q) - F –VC > -F
(P x Q) – VC > 0
(P x Q) > VC
P > VC / Q
P > AVC

Output decision in long run


How about the output decision in the long run?

Since all inputs are variable in the long run, hence, in the long run, all costs are variable. The firm can
eliminate them all by shutting down (so in the long run, all costs are avoidable).

Thus, in the long run, where the firm can avoid all losses by not operating, it pays to shut down if the
firm faces any loss at all.

If

P < LRAC
P x Q < LRAC x Q
TR < TC

Therefore, profit < 0 , so the firm will shut down.

If
P > LRAC
P x Q > LRAC x Q
TR > TC

Therefore, profit > 0, so the firm will produce.


University of London EC2066 (Andy Lee) 46

Tutorial

You might also like